LSAT and Law School Admissions Forum

Get expert LSAT preparation and law school admissions advice from PowerScore Test Preparation.

 Administrator
PowerScore Staff
  • PowerScore Staff
  • Posts: 8929
  • Joined: Feb 02, 2011
|
#27379
Complete Question Explanation
(See the complete passage discussion here: lsat/viewtopic.php?t=11562)

The correct answer choice is (E)

The justification for the correct answer can be found on:

(lines 10-19)

The biologists from line 10 are those who suggest that pathogens can overwhelm a host while still enjoying evolutionary success. This perspective would most be most successfully challenged by correct answer choice (E). If most such pathogens are unable to reproduced in their new hosts, then this destroys the argument of the biologists who claim that replication makes such host incapacitation an evolutionary success for the pathogen. Incorrect answer choices (A) and (D) would both strengthen the author’s argument. Answer choice (B) is irrelevant to the biologists’ argument, as we are discussing those pathogens which do overwhelm or incapacitate their host, and answer choice (C) deals with a different group of pathogens—those which do not overwhelm their hosts, and are thus irrelevant to the strength of the theory presented.
User avatar
 valentina07
  • Posts: 18
  • Joined: Jan 13, 2023
|
#106540
I don't see how answer choice E is "irrelevant." We are discussing pathogens which overwhelm their hosts. This answer choice is talking about pathogens that cannot overwhelm their NEW hosts, not their original ones.

Therefore, the explanation provided isn't very helpful for me.

I was stuck between ACs E and B because I found it difficult to differentiate between them. I found that they both undermined the argument that pathogens that overwhelm their hosts can still find evolutionary success if they can't replicate or overwhelm their new hosts.
User avatar
 valentina07
  • Posts: 18
  • Joined: Jan 13, 2023
|
#106541
**Correction: I don't see how answer choice B is" irrelevant."
User avatar
 Dana D
PowerScore Staff
  • PowerScore Staff
  • Posts: 276
  • Joined: Feb 06, 2024
|
#106567
Hey Valentina,

The question is asking which answer choice would weaken the argument presented in line 10. Here, we learn that the old way of thinking was that pathogens don't overwhelm their hosts. They need hosts to be healthy so they can thrive. The idea we're being asked to look at is the idea that pathogens could achieve evolutionary success if the number of pathogens that can extend into a new host would exceed the number of pathogens lost once the current host is incapacitated. So, it's possible for pathogens to harm their host but still come out as winners.

Answer choice (B) doesn't challenge this idea - it aligns with the new theory no problem. Pathogens don't have to overwhelm their new host - they just have to leave the incapacitated one behind.

Does that make sense?

Get the most out of your LSAT Prep Plus subscription.

Analyze and track your performance with our Testing and Analytics Package.